Από GM !

Συντονιστές: cretanman, Demetres, polysot, socrates, silouan

Άβαταρ μέλους
Ορέστης Λιγνός
Δημοσιεύσεις: 1835
Εγγραφή: Κυρ Μάιος 08, 2016 7:19 pm
Τοποθεσία: Χαλάνδρι Αττικής
Επικοινωνία:

Από GM !

#1

Μη αναγνωσμένη δημοσίευση από Ορέστης Λιγνός » Τρί Μάιος 10, 2016 5:24 pm

Έστω οι θετικοί a,b,c, που είναι τέτοιοι ώστε ab+bc+ca=1.

Να δείξετε ότι \displaystyle\frac{1-a^2}{1+a^2}+\frac{1-b^2}{1+b^2} +\frac{1-c^2}{1+c^2}\leq \frac{3}{2}.


Κερδίζουμε ό,τι τολμούμε!
Άβαταρ μέλους
Ορέστης Λιγνός
Δημοσιεύσεις: 1835
Εγγραφή: Κυρ Μάιος 08, 2016 7:19 pm
Τοποθεσία: Χαλάνδρι Αττικής
Επικοινωνία:

Re: Από GM !

#2

Μη αναγνωσμένη δημοσίευση από Ορέστης Λιγνός » Τρί Μάιος 10, 2016 5:32 pm

Να το προχωρήσουμε λίγο ακόμα ;

Αν ακόμη σας δίνεται ότι a+b+c=3 , αποδείξτε ότι εν τέλει \displaystyle\frac{1-a^2}{1+a^2}+\frac{1-b^2}{1+b^2} +\frac{1-c^2}{1+c^2}\leq 0.

edit : η άσκηση είναι λάθος. Δείτε την ανάρτηση του dement αμέσως μετά.
τελευταία επεξεργασία από Ορέστης Λιγνός σε Τετ Μάιος 11, 2016 9:40 pm, έχει επεξεργασθεί 4 φορές συνολικά.


Κερδίζουμε ό,τι τολμούμε!
dement
Διευθύνον Μέλος
Δημοσιεύσεις: 1417
Εγγραφή: Τρί Δεκ 23, 2008 10:11 am

Re: Από GM !

#3

Μη αναγνωσμένη δημοσίευση από dement » Τρί Μάιος 10, 2016 6:04 pm

orestis26 έγραψε:Έστω οι θετικοί a,b,c, που είναι τέτοιοι ώστε ab+bc+ca=1.

Να δείξετε ότι \displaystyle\frac{1-a^2}{1+a^2}+\frac{1-b^2}{1+b^2} +\frac{1-c^2}{1+c^2}\leq \frac{3}{2}.
Έστω a = \tan A, b = \tan B, c = \tan C με A \geq B \geq C, A, B, C \in (0, \pi / 2). Τότε η συνθήκη γίνεται A + B + C = \pi / 2 ενώ η παράσταση προς μεγιστοποίηση \cos 2A + \cos 2B + \cos 2C.

Λόγω κοιλότητας του \cos x στο (0, \pi/2) έχουμε

\cos 2A + \cos 2B + \cos 2C \leq - \cos (2B + 2C) + 2 \cos (B+C) =

= - 2 \cos^2 (B+C) + 2 \cos(B+C) +1 που μεγιστοποιείται για \cos(B+C) = 1/2 με τιμή 3/2.


Δημήτρης Σκουτέρης

Τα μαθηματικά είναι η μοναδική επιστήμη που θα μπορούσε κανείς να εξακολουθήσει να ασκεί αν κάποτε ξυπνούσε και το σύμπαν δεν υπήρχε πλέον.
Άβαταρ μέλους
Ορέστης Λιγνός
Δημοσιεύσεις: 1835
Εγγραφή: Κυρ Μάιος 08, 2016 7:19 pm
Τοποθεσία: Χαλάνδρι Αττικής
Επικοινωνία:

Re: Από GM !

#4

Μη αναγνωσμένη δημοσίευση από Ορέστης Λιγνός » Τρί Μάιος 10, 2016 6:16 pm

:shock: Η λύση που έχω εγώ είναι με βασικές ανισότητες. Η δικιά σας όμως είναι εκπληκτική! :clap2:


Κερδίζουμε ό,τι τολμούμε!
Άβαταρ μέλους
Ορέστης Λιγνός
Δημοσιεύσεις: 1835
Εγγραφή: Κυρ Μάιος 08, 2016 7:19 pm
Τοποθεσία: Χαλάνδρι Αττικής
Επικοινωνία:

Re: Από GM !

#5

Μη αναγνωσμένη δημοσίευση από Ορέστης Λιγνός » Τετ Μάιος 11, 2016 6:26 pm

Κύριε Σκουτέρη άλλαξα την δεύτερη εκφώνηση. Τι λέτε τώρα ;


Κερδίζουμε ό,τι τολμούμε!
dement
Διευθύνον Μέλος
Δημοσιεύσεις: 1417
Εγγραφή: Τρί Δεκ 23, 2008 10:11 am

Re: Από GM !

#6

Μη αναγνωσμένη δημοσίευση από dement » Τετ Μάιος 11, 2016 7:04 pm

Όχι, ούτε τώρα είναι εντάξει \displaystyle (a = b = 1 - \frac{\sqrt{6}}{3}, c = 1 + \frac{2 \sqrt{6}}{3}). Προσπάθησε να είσαι προσεκτικός, κάποιος μπορεί να φάει ώρες για να αποδείξει κάτι που δεν ισχύει τελικά.


Δημήτρης Σκουτέρης

Τα μαθηματικά είναι η μοναδική επιστήμη που θα μπορούσε κανείς να εξακολουθήσει να ασκεί αν κάποτε ξυπνούσε και το σύμπαν δεν υπήρχε πλέον.
Άβαταρ μέλους
Ορέστης Λιγνός
Δημοσιεύσεις: 1835
Εγγραφή: Κυρ Μάιος 08, 2016 7:19 pm
Τοποθεσία: Χαλάνδρι Αττικής
Επικοινωνία:

Re: Από GM !

#7

Μη αναγνωσμένη δημοσίευση από Ορέστης Λιγνός » Τετ Μάιος 11, 2016 8:35 pm

Ναι έχετε δίκιο . Ξεχάστε λοιπόν το δεύτερο ερώτημα. :oops: :oops:


Κερδίζουμε ό,τι τολμούμε!
Άβαταρ μέλους
Demetres
Γενικός Συντονιστής
Δημοσιεύσεις: 8989
Εγγραφή: Δευ Ιαν 19, 2009 5:16 pm
Τοποθεσία: Λεμεσός/Πύλα
Επικοινωνία:

Re: Από GM !

#8

Μη αναγνωσμένη δημοσίευση από Demetres » Τετ Μάιος 11, 2016 9:31 pm

orestis26 έγραψε:Ναι έχετε δίκιο . Ξεχάστε λοιπόν το δεύτερο ερώτημα. :oops: :oops:
Ορέστη, το σωστό είναι αυτό να δηλωθεί στο δεύτερο ερώτημα. Κάποιοι μπορεί να το δουν και να προσπαθήσουν να το λύσουν πριν να διαβάσουν την ανάρτηση του Δημήτρη ότι είναι λανθασμένο.

Κάνε λοιπόν επεξεργασία στην δεύτερή σου ανάρτηση και πρόσθεσε μια γραμμή που να λέει κάτι του στυλ "Αγνοήστε το ερώτημα μιας και είναι λανθασμένο. Δείτε την ανάρτηση του dement πιο κάτω".


Joaakim
Δημοσιεύσεις: 120
Εγγραφή: Σάβ Φεβ 22, 2020 4:40 pm
Τοποθεσία: Θεσσαλονίκη

Re: Από GM !

#9

Μη αναγνωσμένη δημοσίευση από Joaakim » Τετ Ιουν 02, 2021 9:34 pm

Ορέστης Λιγνός έγραψε:
Τρί Μάιος 10, 2016 5:24 pm
Έστω οι θετικοί a,b,c, που είναι τέτοιοι ώστε ab+bc+ca=1.

Να δείξετε ότι \displaystyle\frac{1-a^2}{1+a^2}+\frac{1-b^2}{1+b^2} +\frac{1-c^2}{1+c^2}\leq \frac{3}{2}.
Παραθέτω και μία λύση χωρίς τριγωνομετρία.

Παρατηρούμε ότι: \displaystyle \frac{1-a^{2}}{1+a^{2}}=\frac{(1+a^{2})-2a^{2}}{1+a^{2}}=1-\frac{2a^{2}}{a^{2}+1}.

Έτσι αρκεί: \displaystyle \sum{\frac{2a^{2}}{a^{2}+1}} \geq 3-\frac{3}{2}=\frac{3}{2} \Rightarrow \sum{\frac{a^{2}}{a^{2}+1} \geq \frac{3}{4}.

Από την ανισότητα Andrescu είναι όμως:

\displaystyle \sum{\frac{a^{2}}{a^{2}+1} \geq \frac{(a+b+c)^{2}}{a^{2}+b^{2}+c^{2}+3}=

\displaystyle =\frac{a^{2}+b^{2}+c^{2}+2(ab+bc+ca)}{a^{2}+b^{2}+c^{2}+3}=\frac{x+2}{x+3},

όπου x=a^{2}+b^{2}+c^{2}.

Αρκεί τώρα: \displaystyle \frac{x+2}{x+3} \geq \frac{3}{4} \Rightarrow 4x+8 \geq 3x+9 \Rightarrow x \geq 1 \Rightarrow

\Rightarrow a^{2}+b^{2}+c^{2} \geq ab+bc+ca,

που προφανώς ισχύει, κι έτσι τελειώσαμε.


Απάντηση

Επιστροφή σε “Άλγεβρα - Θεωρία Αριθμών - Συνδυαστική (Juniors) - Παλαιότερες Συζητήσεις”

Μέλη σε σύνδεση

Μέλη σε αυτήν τη Δ. Συζήτηση: Δεν υπάρχουν εγγεγραμμένα μέλη και 4 επισκέπτες